Đến nội dung

JayVuTF nội dung

Có 63 mục bởi JayVuTF (Tìm giới hạn từ 05-06-2020)



Sắp theo                Sắp xếp  

#541185 Tìm x, y, z thuộc N* biết xyz = xy + yz + zx.

Đã gửi bởi JayVuTF on 18-01-2015 - 15:55 trong Đại số

 

 

 

 

Bài 2: Tìm số tự nhiên x thỏa mãn

 

c) $\frac{1}{3.5}$ + $\frac{1}{5.7}$ + $\frac{1}{7.9}$ +...+ $\frac{1}{\left(2x+1\right)\left(2x+3\right)}$

Đề thiếu dấu = rồi nhỉ

Dạng bài này nhân thêm 2 rồi có công thức Tổng Quát sau $\frac{2}{n(n+2)}=\frac{1}{n}-\frac{1}{n+2}$

Rút gọn nữa là xong




#538703 $c^2=(a-b)^2+4S \left ( \frac{1-\cos C}{\sin C}...

Đã gửi bởi JayVuTF on 21-12-2014 - 19:52 trong Công thức lượng giác, hàm số lượng giác

2) Cho tam giác $ABC$. Cmr:

$c^2=(a-b)^2+4S \left ( \frac{1-\cos C}{\sin C} \right )$

$4S\left ( \frac{1-cosC}{sinC} \right )=\frac{abc}{R}.\frac{R.(c^{2}-(a-b)^{2})}{abc}=c^{2}-(a-b)^{2}$
$\Rightarrow $dpcm



#538702 $c^2=(a-b)^2+4S \left ( \frac{1-\cos C}{\sin C}...

Đã gửi bởi JayVuTF on 21-12-2014 - 19:45 trong Công thức lượng giác, hàm số lượng giác

1) Cho tam giác $ABC$. Cmr:
b) $2\cot B=\cot A+\cot C\Leftrightarrow 2b=a$

 

 

$2\cot B=\cot A+\cot C \Leftrightarrow 2\frac{a^{2}+c^{2}-b^{2}}{4S}=\frac{b^{2}+c^{2}-a^{2}+a^{2}+b^{2}-c^{2}}{4S}$
$\Rightarrow 2a^{2}+2c^{2}-2b^{2}=2b^{2}$
$\rightarrow a^{2}+c^{2}=2b^{2}$
   nếu cho 2b=a thì  thay vào $\Rightarrow 2b^{2}+c^{2}=0$ (vô lý)
$\Rightarrow ..........$



#560773 $8x(2x^2-1)(8x^4-8x^2+1)=1 $

Đã gửi bởi JayVuTF on 21-05-2015 - 19:41 trong Phương trình, Hệ phương trình Lượng giác

có 3 nghiệm: $x=(cos\frac{2\pi }{7}, cos\frac{\pi }{9}, \frac{1}{2})$

Thực ra nếu xét $x\in [-1;1]$ thì pt có tất cả 7 nghiệm: $cos\frac{2\pi }{7}, cos\frac{4\pi }{7}, cos\frac{6\pi }{7}, cos\frac{\pi }{9}, cos\frac{5\pi }{9}, cos\frac{7\pi }{9}, \frac{1}{2}$

Giải cụ thể được không bạn ?




#560510 $8x(2x^2-1)(8x^4-8x^2+1)=1 $

Đã gửi bởi JayVuTF on 20-05-2015 - 15:45 trong Phương trình, Hệ phương trình Lượng giác

Giải PT bằng PP Lượng Giác :

 

$8x(2x^2-1)(8x^4-8x^2+1)=1  , x \in (0;1)$

 

 




#538778 ĐỀ THI HỌC KÌ TRƯỜNG THPT CHU VĂN AN-THÁI NGUYÊN

Đã gửi bởi JayVuTF on 22-12-2014 - 15:19 trong Tài liệu tham khảo khác

Câu 1: (1 điểm) Tìm tập xác định của hàm số sau $y=\frac{x-1}{x^{2}-3x+2}+\sqrt{4-x^{2}}$

 

làm câu dễ nhất vậy

 

$TXD :x^{2}-3x+2 \neq 0 và 4-x^{2} \geq 0 \Leftrightarrow -2\leq x <2 ;x\neq 1$




#548454 Cho a, b, c$> 0$ ; abc=1. Tìm GTLN: $\frac{1...

Đã gửi bởi JayVuTF on 20-03-2015 - 21:51 trong Bất đẳng thức và cực trị

Cho a, b, c$> 0$ ; abc=1. Tìm GTLN: $\frac{1}{a+2b+3}+\frac{1}{b+2c+3}+\frac{1}{c+2a+3}$

$a^2+b^2+c^2\geq 3\sqrt{a^2b^2c^2}=3$
 
Ta có: $a^2+2b+3=a^2+2b+1+2 \geq 2(a+b+1)$
 
Tương tự ta được:
 
$VT \leq \dfrac{1}{2}(\dfrac{a}{a+b+1}+\dfrac{b}{b+c+1} + \dfrac{c}{c+a+1)}$
 
Ta sẽ cm $\dfrac{a}{a+b+1}+\dfrac{b}{b+c+1}+\dfrac{c}{c+a+1} \leq 1$
 
$\Leftrightarrow \dfrac{-b-1}{a+b+1}+\dfrac{-c-1}{b+c+1}+\dfrac{-a-1}{c+a+1} \leq -2$
 
$\Leftrightarrow \dfrac{b+1}{a+b+1}+\dfrac{c+1}{b+c+1}+\dfrac{a+1}{c+a+1} \geq 2$
 
$\Leftrightarrow \dfrac{(b+1)^2}{(b+1)(a+b+1)}+\dfrac{(c+1)^2}{(c+1)(b+c+1)}+\dfrac{(a+1)^2}{(a+1)(c+a+1)} \geq 2$ (*)
 
Theo Cauchy-Schwarz: 
 
$VT(*) \geq \dfrac{(a+b+c+3)^2}{a^2+b^2+c^2+ab+bc+ca+3(a+b+c)+3}$
 
Mà $a^2+b^2+c^2+ab+bc+ca+3(a+b+c)+3\leq \dfrac{1}{2}[a^2+b^2+c^2+2(ab+bc+ca)+6(a+b+c)+9]$
 
$\leq \dfrac{1}{2}(a+b+c+3)^2$
 
$\Rightarrow VT(*) \geq 2=VP(*)$
 
Vậy bđt được cm
 
Nguon HM



#547832 $\frac{a^{2}}{3}+b^{2}+c^...

Đã gửi bởi JayVuTF on 17-03-2015 - 20:51 trong Bất đẳng thức - Cực trị

2,Cho $a,b,c$ thỏa mãn $abc>1$ và $a^{3}>36$

Chứng minh $\frac{a^{2}}{3}+b^{2}+c^{2}> ab+bc+ca$

$VT-VP=\dfrac{a^{2}}{4}+b^{2}+c^{2}-ab-bc+2bc+\dfrac{a^{2}}{12}=(\dfrac{a}{2}-b-c)^{2}+\dfrac{a^{2}-36bc}{12}>0$
$\Rightarrow đpcm $
 
 
Cách khác:
Từ giả thiết suy ra $a>0và bc>0$. Bất đẳng thức cần chứng minh tương đương với
$\dfrac{a^2}{3}+(b+c)^2-3bc-a(b+c)\ge 0\\ \iff \dfrac{1}{3}+\left(\dfrac{b+c}{a}\right)^2-\dfrac{b+c}{a}-\dfrac{3}{a^3}\ge 0$
Vì $a^3>36 nên \dfrac{1}{3}+\left(\dfrac{b+c}{a}\right)^2-\dfrac{b+c}{a}-\dfrac{3}{a^3}> \left(\dfrac{b+c}{a}\right)^2-\dfrac{b+c}{a}+ \dfrac{1}{4}= \left(\dfrac{b+c}{a}-\dfrac{1}{2}\right)^2 >0 $
 

 

Bài này trong VMF có ,trong topic BDT của CD13

 




#569881 $\sum \frac{a}{4b^{2}+1}\ge...

Đã gửi bởi JayVuTF on 04-07-2015 - 15:38 trong Bất đẳng thức và cực trị

Xem tại Đây

 

Nguồn : HM




#569889 $\sum \frac{a}{4b^{2}+1}\ge...

Đã gửi bởi JayVuTF on 04-07-2015 - 16:23 trong Bất đẳng thức và cực trị

Bạn ơi xem lại hộ mình cái. Link vào không được 

Vậy bạn xem tại Đây nhá 




#606508 CMR: $\sum \frac{x}{x+yz}\leq \f...

Đã gửi bởi JayVuTF on 01-01-2016 - 17:11 trong Bất đẳng thức và cực trị

Tham khảo theo cách giải LG : 
 
 
Giải hệ phương trình:
 $\left\{\begin{matrix} x+y+z=1 \hspace{3.9cm}(1)\\ \frac{x}{x+yz} +\frac{y}{y+zx}+\frac{z}{z+xy} = \frac{9}{4} \:\:\:\:\:\:\:(2) \end{matrix} \right.$
Giải:
Nhận thấy $x,y,z=0$ không phải là nghiệm hệ
Viết lại phương trình (1) dưới dạng $\sqrt{\frac{xy}{z}}\sqrt{\frac{xz}{y}}+\sqrt{\frac{yz}{x}}\sqrt{\frac{yx}{z}}+\sqrt{\frac{zx}{y}}\sqrt{\frac{zy}{x}}=1$
Đặt $\sqrt{\frac{xy}{z}}= \tan \frac{A}{2} , \sqrt{\frac{xz}{y}}=\tan \frac{B}{2}, \sqrt{\frac{yz}{x}}=\tan \frac{C}{2}; A,B,C \in(0,\pi)$
ta được $\tan{\frac{A}{2}} \tan{\frac{B}{2}} + \tan{\frac{B}{2}} \tan{\frac{C}{2}}+\tan{\frac{C}{2}} \tan{\frac{A}{2}}=1$
Tương tự như ví dụ trên dễ dàng suy ra $A+B+C= \pi$
Phương trình (2):$\frac{x}{x+yz}+\frac{y}{y+zx}+\frac{z}{z+xy} =\displaystyle \frac{1}{1+tan^2\frac{A}{2}}+\frac{1}{1+tan^2\frac{B}{2}}+\frac{1}{1+tan^2\frac{C}{2}}= \frac{9}{4}$
$\Leftrightarrow \cos^2 \frac{A}{2}+\cos^2 \frac{B}{2}+\cos^2 \frac{C}{2}=\frac{9}{4}$
$\Leftrightarrow \frac{3+\cos A+\cos B+\cos C}{2}=\frac{9}{4}$
$\Leftrightarrow \cos A+ \cos B+\cos C= \frac{3}{2}$ 
$\Leftrightarrow 1-2\sin^2 \frac{A}{2} +2 \cos \frac{B+C}{2} \cos \frac{B-C}{2}= \frac{3}{2}$ 
$\Leftrightarrow 4\sin^2 \frac{A}{2} +2 \sin \frac{A}{2} \cos \frac{B-C}{2}=\frac{3}{2}$ (*) 
$\triangle  ' =4(\cos^2 \frac{B-C}{2}-1) \geqslant 0 $ .Mặt khác $\cos^2 \frac{B-C}{2}-1 \leqslant 0$ 
 Nên (3) $\Leftrightarrow \left\{\begin{matrix} 2\sin \frac{A}{2}=\cos \frac{B-C}{2} \\ \sin \frac{B-C}{2}=0 \end{matrix} \right.$ $\Leftrightarrow A=B=C=\frac{\pi}{3}$ .
Từ đó suy ra $x=y=z=\frac{1}{3} \square$
 

 

Nguồn : VMF

 

 




#561313 $(x+5)\sqrt{x+1}+1=\sqrt[3]{3x+4}$

Đã gửi bởi JayVuTF on 24-05-2015 - 15:11 trong Phương trình - hệ phương trình - bất phương trình

bạn ngó kĩ lại đi $(x+5)\sqrt{x+1}+1 \leq x+2 $ mà

Ờ còn con $1$  bên vế trái nữa . :icon6: 




#561268 $(x+5)\sqrt{x+1}+1=\sqrt[3]{3x+4}$

Đã gửi bởi JayVuTF on 24-05-2015 - 08:30 trong Phương trình - hệ phương trình - bất phương trình

&nbsp; Đk:$x \geq 1$
$(x+5)\sqrt{x+1}+1=\sqrt[3]{3x+4}.1.1 \leq \frac{3x+4+1+1}{3} =x+2 \Leftrightarrow (x+1)^2 \geq (x+5)^2.(x+1) \Leftrightarrow (x+1)(x^2+9x+24) \leq 0 \Leftrightarrow x \leq -1 $
Suy ra: $x=-1$

$\Leftrightarrow (x+1)^2 \geq (x+5)^2.(x+1)$

 

cái này phải là $(x+2)^2\geq (x+5)^2.(x+1) $




#555923 $\cot A+\cot B+\cot C=R.\frac{a^2+b^2+c^2}...

Đã gửi bởi JayVuTF on 23-04-2015 - 21:44 trong Công thức lượng giác, hàm số lượng giác

Cmr:
$\cot A+\cot B+\cot C=R.\frac{a^2+b^2+c^2}{abc}$

Có : $cotA=\dfrac{cosA}{sinA}=\dfrac{\dfrac{b^2+c^2-a^2}{2bc}}{\dfrac{2S}{bc}}=\dfrac{b^2+c^2-a^2}{4S}$
 
$ \rightarrow \cot A+\cot B+\cot C= \dfrac{a^2+b^2+c^2}{4S}=R.\dfrac{a^2+b^2+c^2}{abc}$



#645579 $P = \sqrt{5x^2+xy+3y^2}+ \sqrt{3x^2+xy+5y^2...

Đã gửi bởi JayVuTF on 19-07-2016 - 20:49 trong Bất đẳng thức và cực trị

Cho $x,y > 0$ thỏa mãn $x+y=2016$. Tìm Min : 

$$P = \sqrt{5x^2+xy+3y^2}+ \sqrt{3x^2+xy+5y^2}+\sqrt{x^2+xy+2y^2}+\sqrt{2x^2+xy+y^2}$$




#560158 $\sqrt{1+\sqrt{1-x^2}}(\sqrt{(1+...

Đã gửi bởi JayVuTF on 18-05-2015 - 14:51 trong Phương trình, Hệ phương trình Lượng giác

Giải PT sau  bằng PP Lượng Giác hóa

 

$ \sqrt{1+\sqrt{1-x^2}}(\sqrt{(1+x)^3}-\sqrt{(1-x)^3} )=2+\sqrt{1-x^2}$

 

 




#560465 $\sqrt{1+\sqrt{1-x^2}}(\sqrt{(1+...

Đã gửi bởi JayVuTF on 20-05-2015 - 08:04 trong Phương trình, Hệ phương trình Lượng giác

ĐKXĐ: $ -1 \le x \le 1$

 

Đặt $x= cost ; t \in [0; \pi]$

 

$\Longrightarrow 2\sqrt{1+sint}(\sqrt{2}.sin^3\frac{t}{2}-\sqrt{2}.cos^3\frac{t}{2})=2+sint$

 

$\Leftrightarrow 2.(sin\frac{t}{2}+cos\frac{t}{2}).\sqrt{2}.(sin\frac{t}{2}-cos\frac{t}{2})(1+sin\frac{t}{2}.cos\frac{t}{2})=2+sint$

 

$\Leftrightarrow -\sqrt{2}.cost(2+ \sint )=2+sint$

 

$\Leftrightarrow cost=\frac{-1}{\sqrt{2}} ....$

Là $\Longrightarrow 2\sqrt{1+sint}(\sqrt{2}.cos^3\frac{t}{2}-\sqrt{2}.sin^3\frac{t}{2})=2+sint$

 

Đáp án $cost=\frac{1}{\sqrt{2}} ....$




#575249 Tìm min, max của $P=x^4+y^6+z^8$

Đã gửi bởi JayVuTF on 25-07-2015 - 15:32 trong Bất đẳng thức và cực trị

Cho x,y,z thỏa mãn x+y+z=0, $-1\leq x;y;z\leq 1$

Tìm GTNN, GTLN của biểu thức  $P=x^4+y^6+z^8$

Max 
 
Có : $$x^2(1-x^2)\ge 0 \leftrightarrow x^2 \ge x^4$$
$$y^2(1-y^4)\ge 0 \leftrightarrow y^2 \ge y^6$$
$$z^2(1-y^6)\ge 0 \leftrightarrow z^2 \ge z^8$$
$$\rightarrow  P \le x^2+y^2+z^2= (x+y+z)^2- 2(xy+yz+zx)=-2(xy+yz+zx)$$
Lại có : $$-1 \le x,y,z \le 1 \rightarrow (1-x)(1-y)(1-z)+(1+x)(1+y)(1+z)\ge 0$$ 
$$\leftrightarrow 2+2(xy+yz+zx)\ge 0 \leftrightarrow -2(xy+yz+zx) \le 2$$
 
$$\rightarrow P \le 2$$



#556715 Chứng minh $1+\frac{1}{2}x^2\geqslant cosA...

Đã gửi bởi JayVuTF on 28-04-2015 - 10:00 trong Bất đẳng thức và cực trị

Cho tam giác ABC bất kì, chứng minh rằng:

$1+\frac{1}{2}x^2\geqslant cosA+x(cosB+cosC)$

 
$1+\frac{1}{2}x^2\geqslant cosA+x(cosB+cosC)$
 
$\leftrightarrow x^2-2(cosB+cosC)x-2cosA+2$
 
$\Delta'=(cosB+cosC)^2+2cosA-2=4[cos^2\frac{B+C}{2}.cos^2\frac{B-C}{2}-sin^2\frac{A}{2}]=4sin^2\frac{A}{2}(cos^2\frac{B-C}{2}-1)\leq 0$
 
$\rightarrow af(x)\geq 0  \rightarrow f(x)\geq 0$



#641681 $P = \dfrac{1}{x^2 + y^ 2 + 2} + \dfrac...

Đã gửi bởi JayVuTF on 22-06-2016 - 07:26 trong Bất đẳng thức và cực trị

Cho $x,y,z $ thuộc [0;2] thỏa mãn $x + y + z = 3 $.Tìm Max : 

 $$P = \dfrac{1}{x^2 + y^ 2 + 2} + \dfrac{1}{y^2 + z^ 2 + 2} + \dfrac{1}{z^2 + x^ 2 + 2} + \sqrt{xy} + \sqrt{yz} + \sqrt{zx}$$




#574772 $x^4+2y^4+3z^4$

Đã gửi bởi JayVuTF on 23-07-2015 - 09:28 trong Bất đẳng thức và cực trị

Cho $x,y,z \ge 0 ,x+y+z=3$ .Tìm Min : $x^4+2y^4+3z^4$

 

(Giải = PP Cân bằng hệ số với Holder  :D )




#547895 Đề thi học sinh giỏi trường QL I năm học 2014-2015

Đã gửi bởi JayVuTF on 17-03-2015 - 22:35 trong Tài liệu tham khảo khác

Câu 4:

2. Cho $a,b\in \mathbb{R}$ thỏa mãn: $(2+a)(1+b)=\frac{9}{2}$

Tìm giá trị nhỏ nhất của biểu thức: $P=\sqrt{16+a^{4}}+4\sqrt{1+b^{4}}$

 

P=\sqrt{16+a^{4}}+4\sqrt{1+b^{4}}\geq\sqrt{(4+4)^2+(a^2+(2b)^2)}$

$(2+a)(1+b)=\frac{9}{2}\leftrightarrow a+2b+ab=\frac{5}{2}$
 
$a^{2}+1\ge 2a$
$4b^{2}+1\geq 4b$
$\frac{1}{2}(4a^{2}+b^{2})\geq 2ab$
$\rightarrow \frac{3}{2}(a^{2}+4b^{2})\geq 2(a+2b+ab)-2=3$
$\rightarrow a^{2}+4b^{2}\geq 2$
thay vào  $\rightarrow P\geq 2\sqrt{17}$



#589075 $T = \sum \sqrt{16a^{2} + a^{2}x^...

Đã gửi bởi JayVuTF on 15-09-2015 - 14:42 trong Bất đẳng thức và cực trị

Giả sử các số thực $a , b , c , x , y , z , t$ thoả mãn hệ : $a + b + c + d  = 2 , ax + by + cz + dt = 6$ . Tìm GTNN của biểu thức :

$T = \sqrt{16a^{2} + a^{2}x^{2}} + \sqrt{16b^{2} + b^{2}y^{2}} + \sqrt{16c^{2} + c^{2}z^{2}} + \sqrt{16d^{2} + d^{2}t^{2}}$ 

 

$$T = \sqrt{16a^{2} + a^{2}x^{2}} + \sqrt{16b^{2} + b^{2}y^{2}} + \sqrt{16c^{2} + c^{2}z^{2}} + \sqrt{16d^{2} + d^{2}t^{2}}\geq \sqrt{(4a+4b+4c+4d)^2+(ax+by+cz+dt)^2}=\sqrt{(4.2)^2+6^2}=10$$




#552268 Cho a,b $\epsilon R$ thoả mãn (2+a)(1+b)=$\frac...

Đã gửi bởi JayVuTF on 07-04-2015 - 21:47 trong Bất đẳng thức và cực trị

Cho a,b $\epsilon R$ thoả mãn (2+a)(1+b)=$\frac{9}{2}$
Tìm min: P=$\sqrt{16+a^{4}}+4\sqrt{1+b^{^{4}}}$

$ P=\sqrt{16+a^{4}}+4\sqrt{1+b^{4}}\geq\sqrt{(4+4)^2+(a^2+(2b)^2)}$

$(2+a)(1+b)=\frac{9}{2}\leftrightarrow a+2b+ab=\frac{5}{2}$
 
$a^{2}+1\ge 2a$
$4b^{2}+1\geq 4b$
$\frac{1}{2}(4a^{2}+b^{2})\geq 2ab$
$\rightarrow \frac{3}{2}(a^{2}+4b^{2})\geq 2(a+2b+ab)-2=3$
$\rightarrow a^{2}+4b^{2}\geq 2$
Thay vào   $\rightarrow P\geq 2\sqrt{17}$



#575202 Cho $a,b,c >0, a+b+c=1$.Cm : $5(a^2+b^2+c^2) \ge 6(a^...

Đã gửi bởi JayVuTF on 25-07-2015 - 09:52 trong Bất đẳng thức và cực trị

1. Cho $a,b,c >0, a+b+c=1$.Cm : $5(a^2+b^2+c^2) \le 6(a^3+b^3+c^3)+1$

 

2.Cho $a,b,c >0$ .Cm : $\frac{a^4+b^4+c^4}{ab+bc+ca}+\frac{3abc}{a+b+c}\ge \frac{2}{3}(a^2+b^2+c^2)$